Solve (x-1)(x+5)=0 using zero product property

Answers

Answer 1
the answer is 1 im pretty sure

Related Questions

Help me
pleaseeee
i need help

Answers

Answer:

A=2, B=28

Step-by-step explanation:

A:6:7

8:24:B

A=8÷4

 =2

B=7×4

 =28

Answer:A=2

B=28

Step-by-step explanation:

help me brain pt Part 5

Answers

Answer is A. Adjacent

6457/25 using partial products

Set up using the same way please.

Answers

Answer: 6457/25=258.28

Step-by-step explanation:

Answer:

258.28  Hope this helps! Please mark brainliest it means a lot to me

Step-by-step explanation:

Th=0

H=2

T=5

0=8

tenth=2

hundredth=8

You could get the answer by doing long division/standard algorithm

Someone plz help me :(

Answers

Step-by-step explanation:

option A is correct answer

Answer:

Should be C if not then A

Step-by-step explanation:

1\4-2\5
plese help and thank
you

Answers

Answer:

3/20

Step-by-step explanation:

3/20 I sure, Just trying this one

someone please help me

Answers

Answer:

  (a)  A is steeper; 4/5 > 2/5

Step-by-step explanation:

The slope of A is the coefficient of x, so is 4/5. This eliminates choices B and D.

The line in the graph has a rise of 2 units for each run of 5 units, so has a slope of ...

  slope = rise/run = 2/5

This eliminates choice C.

The correct description of the relationship is the first one:

  Equation A has a steeper slope, because 4/5 is greater than 2/5.

Find the circumference of the
circle. Use 3.14 for it. Do not
round your answer.
C = [ ? ] feet
18 feet
Enter the number that
belongs in the green box.
Hint: C = 2tr

Answers

The answer is abc deft hi

If a company had a gain of 800$ in March , but in April and May , it had a loss of $400 and 900$ respectively , how mich profit or loss did the company have in three months .
Please answer quickly ,.I will mark you as brainlist ... In Hurry Plz

Answers

500 is the answer
500

Answer:

-500

Step-by-step explanation:

i knew the answer

have a great day ;)

test again Add.

3/8+3/8

Answers

Answer:

3/4 or 0.75

Step-by-step explanation:

[tex] \frac{3 }{8} + \frac{3}{8} [/tex]

transform the equation

[tex] \frac{3 + 3}{8} [/tex]

calculate

[tex] \frac{6}{8} [/tex]

simplify

[tex] \frac{3}{4} [/tex]

or 0.75

What is the measurement of this angle?

Answers

The measurement of all angles is 180, so just subtract the other two angles from 180

180 - 59 - 67 = 54 degrees

Answer:

54°

Step-by-step explanation:

All the angle measures of a triangle must add up to equal 180°. So,

59°+67°+x°=180°

x=54°

PLS HELP ASAP ILL GIVE BRAINLKEST THANKS

Answers

Answer:

w>-20

Step-by-step explanation:

Help me please!

Bill is building a fence around his garden. For every 3 feet of fencing, the cost is $21.

Complete the table below showing the length of the fence and the cost.

Answers

Answer:

(cost ---> length)

35 ---> 5

49 ---> 7

56 ---> 8

70 ---> 10

Which equation has the solution x = 5?
Select each correct answer.


18−2x=9

x/5+5=6

32−4x=12

11 + 6x = 22

3x + 1 = 9

25/x+4=9

Answers

25/x+4=9 I believe is the answer

6.
3. Use the clues to determine
the number
I have 36 hundreds, 34 tens, and
7 ones. What number am I?

Answers

The number is 3947

It should be noted that 36 hundreds will be equal to:

= 36 × 100

= 3600

It should be noted that 34 tens will be:

= 34 × 10

= 340

7 ones will be:

= 7 × 1

= 7.

Therefore, the sum will be:

= 3600 + 340 + 7 = 3947

Answer:

3947

Step-by-step explanation:

the graph shown below expresses a radical function that can be written in the form f(x)=a(x+k)^1/n+c. what does the graph tell you about the value of c in this function?

Answers

Answer:

c = 2.

Step-by-step explanation:

When the first part of the function has the value 0,  f(x) will also be 0 if the c wasn't there.

So the value of c is 2.

From the given graph n is a positive odd number.

What is the exponential function?

Exponential function, as its name suggests, involves exponents. But note that, an exponential function has a constant as its base and a variable as its exponent but not the other way round (if a function has a variable as the base and a constant as the exponent then it is a power function but not an exponential function).

From the graph,

All the values of the graph are positive values (even for the negative values of x), but in the left side we can see that the function decreases and is different than the right side.

So this is not an even function, then n must be an odd number (n odd allows us to have negative values for y = f(x) that happen when x + k is negative).

Also, we can see that the function increases, if n was a negative number, like: n = -N

We would have: [tex]f(x)=\frac{a}{(x+k)^{\frac{1}{N} }} +c[/tex]

So in this case x is in the denominator, so as x increases, we would see that the value of y decreases, but that does not happen, so we can conclude that the value of n must be positive.

Then n is a positive odd number.

Therefore, n is a positive odd number.

Learn more about the exponential function here:

brainly.com/question/11487261.

#SPJ7

PARALLEL PERPENDICULAR OR NEITHER OR SAME LINE

Answers

Answer:

parallel, perpendicular are opposite

Answer:

PARALLEL PERPENDICULAR

Step-by-step explanation:

who can solve this equation and get the right answer
solve x-2y=12 if the domain is {-3,-1,0,2,5}

Answers

Answer:

x-2y=12

Solve for y

x-2y=12

Subtract x on both sides

-2y=-x + 12

Divide both sides by -2

y=\frac{1}{2}x - 6y=21x−6

the domain is {-3,-1,0,2,5}

Domain are the set of x values

Range are the set of y values

We are given with domain x, we need to find out y values using the equation y=\frac{1}{2}x - 6y=21x−6

when x=-3 , y=\frac{1}{2}(-3)-6=\frac{-15}{2}=-7.5y=21(−3)−6=2−15=−7.5

when x=-1 , y=\frac{1}{2}(-1)-6=\frac{-13}{2}=-6.5y=21(−1)−6=2−13=−6.5

when x=0 , y=\frac{1}{2}(0)-6=-6y=21(0)−6=−6

when x=2 , y=\frac{1}{2}(2)-6=-5y=21(2)−6=−5

when x=5 , y=\frac{1}{2}(5)-6=\frac{-7}{2}=-3.5y=21(5)−6=2−7=−3.5

Range is {-7.5, -6.5, -6 , -5, -3.5}

Step-by-step explanation:

solve the equation ? 1 equation with 2 variables ? that is not possible.

what you mean is probably find the range.

x - 2y = 12

-2y = -x + 12

2y = x - 12

y = (x - 12)/2 = (1/2)x - 6

x = -3

y = 1/2 × -3 - 6 = -1.5 - 6 = -7.5

x = -1

y = 1/2 × -1 - 6 = -0.5 - 6 = -6.5

x = 0

y = 1/2 × 0 - 6 = -6

x = 2

y = 1/2 × 2 - 6 = 1 - 6 = -5

x = 5

y = 1/2 × 5 - 6 = 2.5 - 6 = -3.5

so, the corresponding range of the function is

{-7.5, -6.5, -6, -5, -3.5}

Need help really quick look at the picture thank you soooooooo,, muchh

Answers

Answer:

The answer would be -7/12

Step-by-step explanation:

Use math.way itll help you tremendously :)

solve the inequality 4x-3=2x+7

Answers

Answer: x=-2

Step-by-step explanation:

Add three to both sides of the equation

4x-3=2x+7

4−3+3=2−7+3

simplify

4=2−4

4−2=2x-4−2

simplify

2=−4

Divide both sides of the equation by the same term

2=−4

2/2=−4/2

=−2

Answer: x = 5

Step-by-step explanation:

Isolate the variable by dividing each side by factors that don't contain the variable.x=5

Read the scenario. 1. The temperature at 6 A.M. was -5°F. By noon the temperature had risen by 5°F. What was the temperature at noon? Represent these values with integers and model them on a number line. + + + 0 1 2 3 4 5 6 7 8 9 10 -10 -9 -8 -7 -6 -5 -4 -3 -2 -1 Read the scenario. 2. A diver was 8 feet below sea level. The diver swam back up 8 feet. What is the location of the diver? Represent these values with integers and model them using integer tiles.​

Answers

Answer:

mm

Step-by-step explanation:

n is an integer greater than 1
Prove algebraically that 10+ n^2 - (n - 2)^2 is always an even number.
Your final line must have, 'always even' as part of the line.

Answers

Answer:

Proof below.

Step-by-step explanation:

Let f(n)=10+ n^2 - (n - 2)^2.

We want to prove f is even for integer n > or = 1.

Let's show f is even for the base case, n=1.

f(1)=10+1^2-(1-2)^2

f(1)=10+1-(-1)^2

f(1)=11-(1)

f(1)=10

f(1) is even.

Let's assume the for some integer k > or = 1 , we have f(k) is even. This means there is some integer m so that f(k)=2m.

Note: f(k)=10+ k^2 - (k - 2)^2.

We want to show f(k+1) is even.

f(k+1)=10+ (k+1)^2 - ([k+1] - 2)^2

f(k+1)=10+ (k+1)^2 - (k-1)^2

f(k+1)=10+ ((k+1)^2 - (k-1)^2)

That difference is a difference of squares. Let's try factoring it.

f(k+1)=10+ (k+1-[k-1])(k+1+[k-1])

f(k+1)=10+ (2)(2k)

Multiply:

f(k+1)=10+4k

Turns out we didn't even need to use our induction hypothesis since 10+4k is 2(5+2k) which makes f(k+1) even given the factor of 2 present.

This proves for all integers n>=1 that f(n) is even.

We could have this without induction then. Let's do that here below this line.

-------------------

10+ n^2 - (n - 2)^2

10+[n^2 - (n - 2)^2]

Factor difference of squares:

10+(n-(n-2))(n+(n-2))

Simplify:

10+(2)(2n-2)

Factor:

2[5+(2n-2)]

Because of the factor of 2, the given expression is always even for integer n.

What are the sums for both of those?

Answers

I don’t think I can help you with this but good luck! And I hope you have an amazing day! :)

help.. What is the domain of the function

Answers

Answer:

i think d

Step-by-step explanation:

I’ll mark brainliest!! Please help:)

Answers

Answer:

E

Step-by-step explanation:

5. INDIRECT MEASUREMENT A lighthouse casts a 128-foot shadow. A nearby lamppost that

measures 5.25 feet casts an 8-foot shadow.

Write a proportion that can be used to determine the height of the lighthouse. What is the height of the lighthouse?

MGSE9-12.G.SRT 5

b. 5.25

a. 1285.25; 195 ft

X 128

8 ; 84 ft

128

c. 5.25" 8 ; 672 ft

X 128

d. 3 5.25; 195 ft

Answers

Using proportions, it is found that the height of the lighthouse is of 84 ft.

This question is solved by proportions, using a rule of three.For a height of 5.25 feet, the shadow is of 8 feet. What is the height when the shadow is of 128 feet?

The rule of three is:

5.25 feet - 8 feet

x feet - 128 feet

Applying cross multiplication:

[tex]8x = 5.25(128)[/tex]

[tex]x = \frac{5.25(128)}{8}[/tex]

[tex]x = 84[/tex]

The height of the lighthouse is of 84 ft.

A similar problem, which also uses proportions, is given at https://brainly.com/question/24372153

Jason was a recent graduate from a university program with a Bachelor of Science in Medical Technology. He will begin a position at the local hospital working 40 hours a week with an hourly rate of $25.50. He will receive a paycheck every two weeks. In a document, calculate the following. Be sure to show your mathematical calculations.
Gross Pay for 1 paycheck
Deductions for federal tax, state tax, and city tax
Deduction for F.I.C.A. or social security tax
Deduction of $117 for medical and dental insurance.
What would Jason’s net monthly income and net annual salary be after taxes and insurance?

Answers

Answer:

Gross Pay for 1 paycheck

Deductions for federal tax, state tax, and city tax hope it helps :)

Step-by-step explanation:

The net monthly income of Jason is $194.68 and the net annual salary is 23324.16.

What is gross pay?

Gross pay of an employee is basically sum of his basic pay, dearness allowance, any other allowance only. Public providend fund, income tax are not deducted from it.

How to find net pay?

1) Gross paycheck=2*40*25.5=$2040.

2) Federal tax=2040*21%=$428.4

3) State tax=25=$25

4) City tax=2040*12%=$244.8.

5) Deductions=2040*12.45=$252.96.

6) Net monthly income=2*(Gross pay-federal tax-state tax-city tax-deductions-deductions for F.IC.A. or social security tax)

=2*[2040-428.4-25-244.8-252.96-117]

=1943.68

7) Net annual salary=12* net monthly income.

=1943.68*12  ( 1year= 12 months)

=$23324.16

Hence the net monthly income of Jason is $194.68 and the net annual salary is 23324.16.

Learn more about gross pay at https://brainly.com/question/11274742

#SPJ2

Use the long division method to find the result when 6x^3+ 23x^2 + 15x + 28 is
divided by 2x + 7

Answers

The answer is:

3x^3+x+4.

Hope this helped!! :)

When ordering pizzas, 5 pizzas cost $60. Does this mean 9 pizzas will cost $100? Write a proportion and use cross products.

Answers

Answer:

108$.

Step-by-step explanation:

A man Buy television set at a price exclusive of sales tax for $2124 if sales tax of 12% is changed how much money does the man pay

Answers

Answer:

2378.88

Step-by-step explanation:

12/100 x 2124

=254.88

2124+254.88=answer

The force, f acting on a chraged object varies inversley to the square of its distance from another charged object. when the two objects are 0.64 meters apart, the force acting on them is 8.2 netwons

Answers

Btw also found a spelling mistake in the question:instead of newtons you put netwons-      lol-

btw  

Rounded to the nearest thousandth would be 4.444 Newtons (same units you solved for k with)

But anyway here you go:

~Jamie
Other Questions
Learning Task 2: Read each statement or question below carefully and fill in the blank(s) with the best answer by choosing the words inside the box. Write your answers in a separate sheet of paper. cation 1 -ide -ine nonmetals O ion ionic compound anion metals root name 1. Any atom or molecule with a net charge, either positive or negative, is known as en 2. An atom that gains one extra electron forms an with a 1- charge. 3. A positive ion, called a is produced when one or more electrons are lost from a neutral atom. 4. Unlike a cation, which is named for the parent atom, an anion is named by taking the of the atom and changing the ending. 5. The name of each anions is obtained by adding the suffix to the root of the atom name. 6. The always form positive ions. 7. on the other hand, form negative ions by gaining electrons. 8. It is very important to remember that a chemical compound must have a net charge of Choose the best answer to complete this sentence. _________________, Americans are usually willing to identify themselves on the liberal conservative spectrum.Group of answer choicesAsking themWhen they askedThey asked themWhen asked 7/8 times 7/9 what is the answer Find the perimeter. Simplify your answer. Mr. Johnston spends 2/3 of every year in Florida how many months does he spend in Florida each year Please HELP!!!!PLSLSLSLSS leading for meaningfulness, self-determination, competence, and progress are behaviors typical of ________ leaders. On January 1, 2021, the Marjlee Company began construction of an office building to be used as its corporate headquarters. The building was completed early in 2022. Construction expenditures for 2021, which were incurred evenly throughout the year, totaled $9,600,000. Marjlee had the following debt obligations which were outstanding during all of 2021: Construction loan, 9% $2,400,000 Long-term note, 8% 3,200,000 Long-term note, 5% 6,400,000 Required: Calculate the amount of interest capitalized in 2021 for the building using the specific interest method. Which circle has a radius that measures 10 units?10D20F10OG20 Hey! How would you punctuate "In the novel, A Separate Peace by John Knowles, ...." I think I know how I'm just checking In the average household, the tv is on for 6. 75 hours a day. How many hours will have passed after 77. 7 years?. 9. Identify the point corresponding to Q.IT'S NOT A. Q' (-3, -4)A. Q (3, 4)B. Q (4, 1)C. Q (1, 4)D. Q (4, 3) write down three integers below 25 with the range of 10 and the mean of 13 Find the slope of line 2, which is parallel to line 1. Line i goes through the points(-13, -6), (7, 15). The ________ hypothesis is the ideology common in the United States that people get the outcomes they deserve. Group of answer choices outline the ways that others can be supported to understand a healthy diet for individuality What was a negative result of increased trade and interactions of cultures during the Middle Ages? a.. It started the end of the feudal system b.. It led to the spread of the Black Death which greatly reduced the population of Europe and Asia c. It led to the increase of new inventions and scientic discoveries d. It caused the Great Schism in the Roman Catholic Church please helpppppppppoppo An "energy bar" contains 26 ggof carbohydrates.How much energy is this in joules? The Forensic Anthropology Facility was founded by whom?